Is 3 Liters equal to 300 mL. Is 3 yards equal to 12 feet. Is 3 tons equal to 9,000 lb

Answers

Answer 1

Answer:

3 Liters = 3000 ml

3 Yards = 12 feet

3 tons = 6000 lb.

Hope that this helps!

Answer 2

Answer:

3 Liters is equal to 3,000 mL

3 Yards is equal to 9 feet

3 Tons is 6,000 lbs

Step-by-step explanation:

Hope this helped!


Related Questions

find the median of the data in the dot plot below. Mass of each rock in Nija's collection​

Answers

Answer:

40

Step-by-step explanation:

The median is the middle number

There are 10 numbers

The middle is between 5th and 6th number

Take the 5th and 6th numbers and average them

(39+41)/2 = 80/2 = 40

The median is 40

Mass of each rock in Nija's collection is 40.

Answer:

The median is the middle number

n=10 numbers

The middle is between 5th and 6th number

Taking the 5th and 6th numbers:

average(39+41)/2 = 80/2 = 40

Based on data from the U.S. Census Bureau, a Pew Research study showed that the percentage of employed individuals ages 25-29 who are college educated is at an all-time high. The study showed that the percentage of employed individuals aged 25-29 with at least a bachelor's degree in 2016 was 40%. In the year 2000, this percentage was 32%, in 1985 it was 25%, and in 1964 it was only 16%.+
What is the population being studied in each of the four years?
a. college educated individuals
b. college educated individuals aged 25-29
c. individuals aged 25-29
d. employed individuals aged 25-29
e. employed individuals

Answers

Answer:

d. employed individuals aged 25-29

Step-by-step explanation:

"Population" in a research study is the comprehensive group that the experimenter or the researcher is interested in.

It is given that US Census Bureau, showed that percentage of the employed individual who are of age group 25 years to 29 years are college educated and is at all time high.

The research study focuses on the specific age group of individuals those who graduated form college or at least have a bachelor degree.

Thus the population of the research study those who studied in each of the four years are the employed individuals aged from 25-29.

An ice cube is melting, and the lengths of its sides are decreasing at a rate of 0.8 millimeters per minute At what rate is the volume of the ice cube decreasing when the lengths of the sides of the cube are equal to 18 millimeters? Give your answer correct to the nearest cubic millimeter per minute. Rate of decrease: millimeters3 per minute.

Answers

Answer:

The rate of decrease is: [tex]43.2mm^3/min[/tex]

Step-by-step explanation:

Given

[tex]l = 18mm[/tex]

[tex]\frac{dl}{dt} = -0.8mm/min[/tex] ---- We used minus because the rate is decreasing

Required

Rate of decrease when: [tex]l = 18mm[/tex]

The volume of the cube is:

[tex]V = l^3[/tex]

Differentiate

[tex]\frac{dV}{dl} = 3l^2[/tex]

Make dV the subject

[tex]dV = 3l^2 \cdot dl[/tex]

Divide both sides by dt

[tex]\frac{dV}{dt} = 3l^2 \cdot \frac{dl}{dt}[/tex]

Given that: [tex]l = 18mm[/tex] and [tex]\frac{dl}{dt} = -0.8mm/min[/tex]

[tex]\frac{dV}{dt} = 3 * (18mm)^2 * (-0.8mm/min)[/tex]

[tex]\frac{dV}{dt} = 3 * 18 *-0.8mm^3/min[/tex]

[tex]\frac{dV}{dt} = -43.2mm^3/min[/tex]

Hence, the rate of decrease is: 43.2mm^3/min

what is
3⋅f(−4)−3⋅g(−2)=

Answers

Answer:

[tex]3 * f(-8) - 3 * g(-2) = 6[/tex]

Step-by-step explanation:

Given

This question has a missing graph (See online)

Required

[tex]3 * f(-8) - 3 * g(-2)[/tex]

From the graph:

[tex]f(-8) = -2[/tex]

and

[tex]g(-2) = -4[/tex]

So, we have:

[tex]3 * f(-8) - 3 * g(-2) = 3 * -2 -3 * -4[/tex]

[tex]3 * f(-8) - 3 * g(-2) = 6[/tex]

please help me!!!!!!!!!!!!!!!!!!!!!!!!!!!!!

Answers

9514 1404 393

Answer:

  the correct answer is marked

Step-by-step explanation:

The circle with center (h, k) and radius r has equation ...

  (x -h)² +(y-k)² = r²

For the given center, that is ...

  (x -1)² +(y +1)² = r²

The value of r² can be found by using the given point for x and y, and seeing what value of r² makes the equation true.

  (5 -1)² +(7 +1)² = 4² +8² = 16 +64 = 80

So, r² = 80 makes the equation true, and that equation is ...

  (x -1)² +(y +1)² = 80

Simplify: -8x +7-5-5x + 5x
A
- 8x + 2
B
-18x+12
-x-5
D
-X+12

Answers

Answer:

A) -8x+2

Step-by-step explanation:

-8x + 7 - 5 - 5x + 5x

-8x -5x + 5x + 7 - 5

- 8x+2

Which of the following is most likely the next step in the series?

Answers

Answer:

B

Step-by-step explanation:

Hi there!

TL;DR: Observe the vertices of the shapes inside the circles and their relationship with the circle.

For the first figure, the rectangle has 4 vertices and there are 4 dots on the perimeter of the circle.

For the second figure, the triangle has 3 vertices and there are 3 dots on the perimeter of the circle.

For the third figure, the line has 2 points and there are 2 dots on the perimeter of the circle.

For the fourth figure, there would most likely be only one dot on the perimeter of the circle (4, 3, 2, 1). The only option that shows this is B.

I hope this helps!

Right answers only pls I am gonna fail rn pls help

Answers

Answer:

y = 5x – ¼

Step-by-step explanation:

The equation for the slope intercept form is given by:

y = mx + c

Where m and c are the slope and intercept on the y-axis respectively.

From the equation i.e y = mx + c, we can see that y is the subject of the equation.

Now, comparing the equation of the slope intercept form (i.e y = mx + c) with those given in the question above, we can see that only the equation:

y = 5x – ¼

has y as the subject of the equation.

Therefore, y = 5x – ¼ gives the correct answer to the question.

Find the amount of interest for a 18-year investment of $4300 at a simple annual rate
of 4.07%
PLEASE ANSWE QUICK

Answers

1 year will earn 4300 x 4.07% = $175.01
18 years will earn 175.01 x 18 = $3150.18

Suppose annual salaries for sales associates from Geoff's Computer Shack have a mean of $32,500 and a standard deviation of $2,500.
a. Calculate and interpret the z-score for a sales associate who makes $36,000.
b. Suppose that the distribution of annual salaries for sales associates at this store is bell-shaped. Use the empirical rule to calculate the percentage of sales associates with salaries between $27,500 and $37,500.
c. Use the empirical rule to determine the percentage of sales associates with salaries less than $27,500.
d. Still suppose that the distribution of annual salaries for sales associates at this store is bell-shaped. A sales associate makes $42,000. Should this salary be considered an outlier? Explain.

Answers

Answer:

The answer is below

Step-by-step explanation:

Given that mean (μ) = $32500, standard deviation (σ) = $2500.

a) The z score is used to determine by how many standard deviations the raw score is above or below the mean. The z score is given by:

[tex]z=\frac{x-\mu}{\sigma}[/tex]

For x < 36000:

[tex]z=\frac{36000-32500}{2500}=1.4[/tex]

From the normal distribution table, P(x < 36000) = P(z < 1.4) = 0.9192 = 91.92%

b) One standard deviation of mean = μ ± σ = (32500 ± 2500) = (30000, 35000)

Two standard deviation of mean = μ ± 2σ = (32500 ± 2*2500) = (27500, 37500)

Empirical rule states that 68% of data falls within one standard deviation from the mean, 95% falls within two standard deviation from the mean and 99.7% falls within one standard deviation from the mean.

Hence 95% of salaries is between $27,500 and $37,500.

c) 95% of salaries is between $27,500 and $37,500.

P(x < 27500) = (100% - 95%) / 2 = 2.5%

d) If the z score is less than -3 or greater than 3, it is considered an outlier.

For x < 42000:

[tex]z=\frac{42000-32500}{2500}=3.8[/tex]

Hence $42000 is an outlier

uppose that grade point averages of undergraduate students at one university have a bell-shaped distribution with a mean of 2.5 and a standard deviation of 0.37. Using the empirical rule, what percentage of the students have grade point averages that are no more than 3.24

Answers

Answer:

[tex]P(x \le 3.24) = 0.97725[/tex]

Step-by-step explanation:

Given

[tex]\bar x = 2.5[/tex]

[tex]\sigma = 0.37[/tex]

Required

Percentage that is not more than 3.24

The above implies that:

[tex]x = 3.24[/tex]

Calculate z score

[tex]z = \frac{x - \bar x}{\sigma}[/tex]

[tex]z = \frac{3.24 - 2.5}{0.37}[/tex]

[tex]z = \frac{0.74}{0.37}[/tex]

[tex]z = 2[/tex]

So, the probability is represented s:

[tex]P(x \le 3.24) = P(z \le 2)[/tex]

From z probability

[tex]P(z \le 2) = 0.97725[/tex]

Hence:

[tex]P(x \le 3.24) = 0.97725[/tex]

Complete the factored form of the linear expression.
15x + 12y = ( )( + )

Answers

Answer:

3(5x + 4y)

Step-by-step explanation:

Given

15x + 12y ← factor out 3 from each term

= 3(5x + 4y) ← in factored form

3(5x+4y) both 15 and 12 have the common dividend of 3 therefore you take it out of the equation :)

Solve the homogeneous linear system corresponding to the given coefficient matrix.

[1 0 0 1]
[0 0 1 0]
[0 0 0 0]

(x1, x2, x3, x4) =________

Answers

This question is incomplete, the complete question is;

Solve the homogeneous linear system corresponding to the given coefficient matrix. (If there is no solution, enter NO SOLUTION. If the system has an infinite number of solutions, set x4 = t and x2 = s and solve for x1 and x3 in terms of t and s.)

[1 0 0 1]

[0 0 1 0]

[0 0 0 0]

(x1, x2, x3, x4) =________

Answer:

the solution for the given system is; ( x₁, x₂, x₃, x₄ ) = ( -t, s, 0, t )

Step-by-step explanation:

Given the data in the question;

coefficient matrix

[tex]\left[\begin{array}{cccc} 1&0&0&1 \\ 0&0&1&0 \\ 0&0&0&0 \end{array}\right][/tex]

Now, from linear system;

[tex]\left[\begin{array}{cccc} 1&0&0&1 \\ 0&0&1&0 \\ 0&0&0&0 \end{array}\right] \left[\begin{array}{ccc}x_1\\x_2\\x_3\\x_4\end{array}\right] = \left[\begin{array}{ccc}0\\0\\0\\0\end{array}\right][/tex]

So, with the matrix, the associated equation is;

x₁ + x₄ = 0, x₃ = 0

Number of variables is 4 and ranked of the matrix is 2,

Hence, there are infinite solutions,

There are also two free variables;

from the question,

Let x₄ = t and x₂ = s be the free variables

so

x₁ + x₄ = 0

x₁ + t = 0

x₁ = -t

Therefore, the solution for the given system is;

( x₁, x₂, x₃, x₄ ) = ( -t, s, 0, t )

The product of a and the sum of a and 34?

Answers

A times 34 idfbutcgnjytfxvhitd

Consider the function f(x) = 2^x
and function g
g(x) = f(x) + 6
How will the graph of function g differ from the graph of function ?

Answers

Answer:

The graph of function g is the graph of function f shifted 6 units up

Step-by-step explanation:

If you plug in the values, [tex]g(x) = 2^{x} + 6[/tex]. If the 6 was added or subtracted from the x in the exponent, it would shift horizontally (left and right), but adding 6 to f(x) separately moves the graph vertically (up and down). Hope this helps.

A ball thrown in the air has a height of y = - 16x² + 50x + 3 feet after x seconds. a) What are the units of measurement for the rate of change of y? b) Find the rate of change of y between x = 0 and x = 2?

Answers

Answer:

(a) ft/s

(b) 1ft/s

Step-by-step explanation:

Given equation;

y = (- 16x² + 50x + 3)ft       -------------(i)

Where;

y is measured in feet(ft)

x is measured in seconds(s).

(a) The rate of change of y with respect to x is found by dividing the total change in y by the total change in x. i.e

Δy / Δx

Where;

Δy = y₂ - y₁  

Δx = x₂ - x₁

∴ Δy / Δx = [tex]\frac{y_2 - y_1}{x_2 - x_1}[/tex]              --------------(ii)

Since y is measured in feet, Δy will also be measured in feet.

Also, since x is measured in seconds, Δx will also be measured in seconds.

Therefore, the rate of change of y with respect to x (Δy / Δx) will be measured in feet per second (ft/s)

(b) The rate of change of y between x  = 0 and x = 2 can be found by using equation (ii)

Where;

y₂ is the value of y at x = 2 found by substituting x = 2 into equation (i)

y₁ is the value of y at x = 0 found by substituting x = 0 into equation (i)

=> y₂ =  - 16(2)² + 50(2) + 3 = 39

=> y₁ =  - 16(1)² + 50(1) + 3 = 37

Now, substitute the values of y₂, y₁, x₂ and x₁  into equation (ii)

Δy / Δx = [tex]\frac{39 - 37}{2 - 0}[/tex]  

Δy / Δx = [tex]\frac{2}{2}[/tex]

Δy / Δx = 1

Therefore, the rate of change of y is 1 ft/s

 

What is the value of x?
Enter your answer as a decimal in the box. Round only your final answer to the nearest tenth.
x =___m

Answers

Answer:

<W=180 - (30+81)

<W=69°

Using Sine rule to evaluate x

x/sin30 = 19/sin69

x= 19sin30/sin69

x= 10.2m ( Nearest tenth)


help please quick please

Answers

Answer:

the answer is 3.5

Step-by-step explanation:

I need help, please answer A and B.

Answers

Answer:

a. 81 square inches

b. 108 inches

Step-by-step explanation:

The area of all 5 squares combined is 405 square inches. This is 5 times the area of 1 square. 405/5 = 81. The area of 1 square is 81 square inches.

Since this is a square, the sides are equal. Thankfully the area of 1 square is a perfect square so the length of one side of the square is a full number, which is 9. All of the squares are congruent as shown in the diagram so we simply add every length together (there are 12 sides, each side is 9 inches).

12*9 = 108 and that is the perimeter.

Answer:

area

Step-by-step explanation:

a = 405/5 = 81

a =81 square inches

perimeter

p = √81

= 9

=4×9×5

=180 inches

What is the value of x?
O 20
(x + 40)
(3x)
R
O 35
w
O 60
S
070

Answers

Answer:

Hello! answer: 20

Step-by-step explanation:

These are vertical angles meaning they will have the same measure so... 20 + 40 = 60 20 × 3 = 60 therefore x = 20 hope that helps!

In the given scenario, the value of x is 20. The correct option is A.

What is parallelogram?

A parallelogram is a quadrilateral with two pairs of parallel sides. This means that opposite sides of a parallelogram are parallel and congruent, and opposite angles are congruent as well.

To find the value of x in the given diagram, we need to use the fact that the opposite sides of a parallelogram are equal in length.

Therefore, we can set the expressions for the opposite sides equal to each other and solve for x:

x + 40 = 3x

Simplifying the equation, we get:

40 = 2x

Dividing both sides by 2, we get:

x = 20

Therefore, the value of x is 20.

For more details regarding parallelogram, visit:

https://brainly.com/question/29147156

#SPJ7

How do I do this ahh!?

Answers

Just scan it, there’s an answer for it already I’m pretty sure

which point lies on the line described by the equation below?
( I am pulling an all nighter for high school and this question is really important)

Answers

Answer:

F (5,-8)

Step-by-step explanation:

By rearranging,

y= 4x-28

Substitute each of the x values of the answers, eventually you will get to F and discover that when x = 5, y= -8

y= 4(5) - 28

y= -8

So, when x= 5, y= -8,

Point F is  (5,-8)

1)Light travels at the speed of 3,00,000 km in one second ,if it has
already travelled 1,05,342 km, how much more distance is left for the
ray of light to travel?

Answers

Answer:

1,94,658km

Step-by-step explanation:

If you are talking about how much distance it has to travel in 1 second then you can simply subtract 1,05,342 from 3,00,000. This process gives total of 1,94,658km remaining for it to travel.

Goodluck

The hours of daylight, y, in Utica in days x from January 1, 2013 can be modeled by the equation y = 2.89sin(0.0145x-1.40) + 10.99. How many hours of daylight, to the nearest tenth, does this model predict for February 21, 2013?

Answers

Answer:

9.3 hours

Step-by-step explanation:

Given

[tex]y = 2.89\sin(0.0145x-1.40) + 10.99.[/tex]

Required

Hours of sunlight on Feb 21, 2013

First, calculate the number of days from Jan 1, 2013 to Feb 21, 2013

[tex]days = 52[/tex]

So:

[tex]x = 52[/tex]

So, we have:

[tex]y = 2.89\sin(0.0145x-1.40) + 10.99.[/tex]

[tex]y = 2.89\sin(0.0145*52-1.40) + 10.99.[/tex]

[tex]y = 2.89\sin(0.754-1.40) + 10.99.[/tex]

[tex]y = 2.89\sin(-0.646) + 10.99.[/tex]

[tex]y = 2.89*-0.6020 + 10.99.[/tex]

[tex]y = -1.740 + 10.99.[/tex]

[tex]y = 9.25[/tex]

[tex]y = 9.3[/tex] --- approximated

Which statement correctly identifies the line of reflection?


The triangles are reflected across the x-axis.
The triangles are reflected across the y-axis.
The triangles are reflected across the line y = x.
The triangles are reflected across the line y = –x.

Answers

The last on in the bottom

12 ducks fly overhead. Each of 6 hunters picks one duck at random to aim at and kills it with probability 0.6. What's the expected number of hunters who hit the duck they aim at?

Answers

Answer:

The expected number of hunters who hit the duck they aim at is 3.6

Step-by-step explanation:

Given;

number of hunters, n = 6

the probability of killing a duck, p = 0.6

The expected number of hunters who hit the duck they aim at?

In binomial distribution, the expected value is equal to the product of the number of trials and the probability of success.

The expected number of hunters who hit the duck they aim at is calculated as follows;

E = np

E = 0.6 x 6

E = 3.6

Therefore, the expected number of hunters who hit the duck they aim at is 3.6

A survey of 2392 adults ages 18 and over asked what type of food they would be most likely to choose at a restaurant. The results are shown in the figure.

A circle graph titled Survey Results is divided into 6 regions showing different types of restaurants. Starting from the top right, the regions are labeled, American 670, Italian 526, Mexican 407, Chinese 383, Japanese 167, Other 239.

What is the probability that an adult chosen at random prefers Italian food? Round your answer to the nearest whole percent.

Answers

Answer:

22%

Step-by-step explanation:

their is a total of 2392 adults, the number of people, who would be most likely to chose Italian food is 526.

p= 526/ 2392=22%

The probability of choosing Italian food is 21.9%.

What is probability?

"Probability means possibility. It is a branch of mathematics that deals with the occurrence of a random event. The value is expressed from zero to one".

For the given situation,

Total number of adults = 2392

American food = 670

Italian food = 526

Mexican food = 407

Chinese food = 383

Japanese food = 167

Others = 239

The event is the probability of choosing Italian food.

[tex]P(e)=\frac{526}{2392}[/tex]

⇒[tex]P(e)=0.219[/tex]

Rounded to nearest whole percent = [tex]0.219[/tex] × [tex]100[/tex]

⇒[tex]21.9\%[/tex]

Hence we can conclude that the probability of choosing Italian food is 21.9%.

Learn more about probability here

brainly.com/question/13604758

#SPJ2

Find the angle(s) of intersection between the equations f (x) = x^2 + 2x + 1 and g(x) = 1.

There could be more that one answer

18.4
26.6
71.6
63.4 (already know that this is definitely one of the options)

Answers

Answer:

seems to be the same as the other angle

Step-by-step explanation:

i use desmos for these graphs btw

A video game that usually costs $30.65 is marked down 60%. Kelvin determined that the new price of the game would be $18.39. Look at Kelvin's work and find his error. ($30.65)(0.60) = $18.39

*Please give an explanation longer than 1 sentence :,)

Answers

Answer:

Kelvin’s error is that when he got the final result, that was the amount that was marked down. He still needed to find the price after the original price was marked down by that number wich in this case is $18.39. So using one of his steps, 30.65(0.6)=18.39. We can subtract 30.65 (original price) and 18.39 (mark down price). You’d get 11.26 dollars as the final price.

What is the probability that you roll a dice and get 3, then get an ace from a deck, then (without replacing the other card) draw a black 2?

I Really need help!

Answers

Answer:  1/1989

=============================================================

Explanation:

1/6 is the probability of rolling a '3' since there is one side we want out of 6 total.

The probability of getting an ace is 4/52 because there are 4 aces out of 52 cards total.

If that card isn't put back, then we have 52-1 = 51 cards left. That means the probability of drawing a black 2 is 2/51. The numerator 2 has nothing to do with the card value of '2'. It has to do with the fact we have 2 black cards (2 of spades, 2 of clubs) out of 51 left over.

We multiply the fractions to get the final answer

(1/6)*(4/52)*(2/51)

(1*4*2)/(6*52*51)

8/15912

1/1989

When using a calculator, we can see that,

1/1989 = 0.0005 = 0.05%

There is a very small chance of all three events happening at the same time.

Other Questions
Who was the first person to suggest the existence of atoms? Read the following excerpt from Eleanor Roosevelts speech What Libraries Mean to the Nation.I know one place in the northern part of the state where I camped for a while in the summer, and I went to the school and talked to the teachers. They are using school books which have been passed down from one child to another. They have practically no books outside of the textbooks. The children in the district are so poor and some of them so pathetic that I suppose the struggle to live has been so great you could not think much about what you fed the mind, but I came away feeling that right there, in one of the biggest and richest states in the country, we had a big area that needed books and needed libraries to help these schools in the education of the children, and, even more, to help the whole community to learn to live through their minds.We are doing a tremendous amount through the home economics colleges to help people to learn how to live in their homes, to better their standards of material living. We have got to think in exactly the same way about helping them to live mentally and to attain better standards, and we can do it only through the children. We can do ground work with the children; we must begin with them; but we have got to do a tremendous amount with the older people.What message is Roosevelt trying to convey?A. There needs to be greater effort put into teaching children how to read.B. Parents and children need to be educated about the value of books.C. Libraries are necessary because they improve peoples overall quality of life.D. A higher priority is needed on learning rather than material concerns. Ninety-nine percent of the oxygen reaching cells can be bestrepresented by Please help the question is in the picture. At the beginning of June, Circuit Country has a balance in inventory of $2,050. The following transactions occur during the month of June. June 2 Purchase radios on account from Radio World for $1,750, terms 2/15, n/45. June 4 Pay cash for freight charges related to the June 2 purchase from Radio World, $210. June 8 Return defective radios to Radio World and receive credit, $200. June 10 Pay Radio World in full. June 11 Sell radios to customers on account, $3,100, that had a cost of $2,250. June 18 Receive payment on account from customers, $2,100. June 20 Purchase radios on account from Sound Unlimited for $2,850, terms 2/10, n/30. June 23 Sell radios to customers for cash, $4,350, that had a cost of $2,650. June 26 Return damaged radios to Sound Unlimited and receive credit of $500. June 28 Pay Sound Unlimited in full.Required:a. Assuming that Circuit Country uses a perpetual inventory system, record transactions using the following account titles: Cash, Accounts Receivable, Inventory, Accounts Payable, Sales, and Cost of Goods Sold.b. Prepare the top section of the multiple-step income statement through gross profit for the month of June. Which variable is represented by the following symbol? a. Wavelengthb. Amplitudec. Frequencyd. Velocity A carpenter applies a force of 60N horizontally to push a plane 40 cm along a piece of wood, how much work does she do? Beth broke her ankle bone. The doctor gives her a brochure on foods she can eat to help restore thehealth of herbones. Which element will help Bethstrengthen the bone? please answer thank you (x^2+2x-1)+(2x^2 2x+1) help urgent pleasethe graphs below have the same shape. what is the equation of the blue graph?no links please I really extreme help Need help with number one please Paul rolls a fair dice 174 times.How many times would Paul expect to roll an odd number? Which literary elements define narrative poetry? Select 4 options. Kathleen hiked to the top of a mountain. She started the hike at 7:25 A.M. It took 3 hours and 45 minutes to get to the top of the mountain and 1 hour and 30 minutes to hike back down. What time was it when Kathleen finished the hike? A museum conducts a survey of its visitors in order to assess the popularity of a device which is used to provide information on the museum exhibits. The device will be withdrawn if fewer than 20% of all of the museums visitors make use of it. Of a random sample of 100 visitors, 15 chose to use the device.i. Carry out an appropriate hypothesis test at the 5% significance level tosee if the device should be withdrawn and state your conclusions.ii. Calculate the p-value of the test What is the least number of apples that can be shared equally among either 6, 10 or 15 children? A pair of shoes that normally cost $24 is 25% off. what is the selling price of the jeans? Astronauts use a tool in space that weighs 5 ounces on Earth. How many grams does the tool weigh? Round your answer to the nearest hundredth.